• Non ci sono risultati.

Tutorato di Analisi 2 - AA 2014/15

N/A
N/A
Protected

Academic year: 2021

Condividi "Tutorato di Analisi 2 - AA 2014/15"

Copied!
14
0
0

Testo completo

(1)

Tutorato di Analisi 2 - AA 2014/15

Emanuele Fabbiani 6 maggio 2015

1 Estremi vincolati.

1.1 Ricerca di estremi vincolati.

Per risolvere gli esercizi di ottimizzazione vincolata è innanzitutto essenziale prestare attenzione al vincolo:

• se questo è rappresentato da un'equazione, allora è suciente studiare il comportamento della funzione sulla curva descritta dal vincolo mediante uno dei tre metodi che verranno presentati;

• se invece appare sotto forma di disequazione, è necessario dapprima studiare l'andamento della funzione nei punti interni all'insieme (vedi sezione Massimi e Minimi) e poi considerare la frontiera, come nel punto precedente.

Esistono tre metodi di ottimizzazione vincolata:

1. Metodo parametrico: da utilizzare quando il vincolo è una curva di cui si conosce la parametrizzazione in funzione di una sola variabile (vedi sezione Curve): ne sono esempi una retta, una parabola, una circonferenza,...

2. Metodo dei moltiplicatori di Lagrange: richiede ipotesi di regolarità sia sulla funzione che sul vincolo, che solitamente vengono date per scontate ma soprattutto esige che il vincolo - o i vincoli - possano essere scritti nella forma g (x, y) = 0.

3. Metodo graco: richiede che si possano disegnare con facilità le curve di livello della funzione in esame.

Per brevità, nelle soluzioni verrà impiegato solo il metodo che, di volta in volta, risulta più semplice. Nulla vieta di tentare, come ulteriore esercizio, di seguire altre strade.

Determinare gli eventuali punti di massimo e minimo della funzione f soggetta al vincolo I.

1.

f (x, y) = x2+ y − 1 I = f rontiera del triangolo di vertici A (1; 0) B (0; 1) O (0; 0) Si disegna innanzitutto il vincolo:

0.0 0.1 0.2 0.3 0.4 0.5 0.6 0.7 0.8 0.9 1.0 0.0

0.1 0.2 0.3 0.4 0.5 0.6 0.7 0.8 0.9 1.0

x y

Figura 1.1: Vincolo.

(2)

La funzione è continua e l'insieme ammissibile è chiuso e limitato. Per Weierstass esistono massimo e minimo assoluti. Si valutano le tre possibili opzioni:

(a) Metodo parametrico: ottimo, il vincolo è composto dall'unine di più segmenti, facilmente para- metrizzabili.

(b) Metodi dei moltiplicatori di Lagrange: pessimo, il vincolo non è esprimibile nella forma g (x, y) = 0.

(c) Metodo graco: buono, le curve di livello sono parabole e l'insieme è un triangolo.

Si propende per il METODO PARAMETRICO.

Si prende in considerazione il segmento OA e si sceglie una parametrizzazione:

γ (t) = (t; 0) , t ∈ [0; 1] (1.1)

Si sostituiscono le coordinate della curva γ all'interno della funzione f, ottenendo una funzione di una sola variabile:

h (t) = [f (x, y)]OA=x2+ y − 1 x = t y = 0

= t2− 1 (1.2)

Si calcolano i punti stazionari di h:

h0(t) = 2t = 0 (1.3)

t0= 0 (1.4)

A cui corrispondono le coordinate:

(x0= t0= 0

y0= 0 (1.5)

Ed il valore della funzione:

f (x0, y0) =x2+ y − 1 x = 0 y = 0

= −1 (1.6)

Si considerano anche gli estremi del dominio di t: t0= 0e t1= 1. Il primo è già stato preso in esame; per quanto riguarda il secondo:

(x1= t1= 1

y1= 0 (1.7)

f (x1, y1) =x2+ y − 1 x = 1 y = 0

= 0 (1.8)

I primi due candidati per il confronto nale sono quindi P0(0; 0)e P1(1; 0). Si prende poi in considerazione il segmento OB e si sceglie una parametrizzazione:

γ (t) = (0; t) , t ∈ [0; 1] (1.9)

Si sostituiscono le coordinate della curva γ all'interno della funzione f, ottenendo una funzione di una sola variabile:

h (t) = [f (x, y)]OA=x2+ y − 1 x = 0 y = t

= t − 1 (1.10)

Si calcolano i punti stazionari di h:

h0(t) = 1 = 0 (1.11)

L'equazione è impossibile: non esistono punti stazionari. Si considerano anche gli estremi del dominio di t: t0= 0e t2= 1. Il primo è già stato preso in esame; per quanto riguarda il secondo:

(x2= 0

y2= t2= 1 (1.12)

f (x1, y1) =x2+ y − 1 x = 0 y = 1

= 0 (1.13)

(3)

Un altro candidato è quindi P2(0; 1).

Si prende passa inne al segmento AB e si sceglie una parametrizzazione:

γ (t) = (t; −t + 1) , t ∈ [0; 1] (1.14)

Si sostituiscono le coordinate della curva γ all'interno della funzione f, ottenendo una funzione di una sola variabile:

h (t) = [f (x, y)]OA=x2+ y − 1 x = 0 y = t

= t2− t + 1 − 1 = t2− t (1.15)

Si calcolano i punti stazionari di h:

h0(t) = 2t − 1 = 0 (1.16)

t3=1

2 (1.17)

A cui corrispondono le coordinate:

(x3= t3= 12

y3= −t3+ 1 = 12 (1.18)

Ed il valore della funzione:

f (x0, y0) =x2+ y − 1 x = 14 y = 14

= −1

4 (1.19)

Il punto P3 1 2; 12

è quindi l'ultimo candidato: gli estremi t = 0 e t = 1 corrispondono infatti ai punti P0

e P1già in elenco.

Ora il confronto nale: tra tutti i candidati individuati, quello cui corrisponde il valore più alto della funzione è il massimo, quello cui corrisponde il valore più basso il minimo.

punto x y f (x, y)

P0 0 0 −1 min

P1 1 0 0 M AX

P2 0 1 0 M AX

P3 1 2

1 214

(1.20)

2.

f (x, y) = x2+ 3y I =(x; y) ∈ R2: 9x2+ 4y2= 36 Si cerca innanzitutto di individuare la curva descritta dal vincolo:

9x2+ 4y2= 36 (1.21)

9x2 36 +4y2

36 = 36

36 (1.22)

x2 4 +y2

9 = 1 (1.23)

Un ellisse con i fuochi sull'asse y, centro nell'origine e vertici (±2; 0) e (0; ±3).

−2 −1 1 2

−3

−2

−1 1 2 3

x y

Figura 1.2: Vincolo.

(4)

La funzione è continua e l'insieme ammissibile è chiuso e limitato. Per Weierstass esistono massimo e minimo assoluti. Si valutano le tre possibili opzioni:

(a) Metodo parametrico: ottimo, il vincolo è una curva facilmente parametrizzabile.

(b) Metodi dei moltiplicatori di Lagrange: ottimo, il vincolo è esprimibile nella forma g (x, y) = 9x2+ 4y2− 36 = 0.

(c) Metodo graco: buono, le curve di livello sono parabole e l'insieme è un ellisse.

Si propende per il METODO PARAMETRICO.

Si parametrizza l'ellisse come proposto nella sezione Curve:

γ (t) = (a cos (t) ; b sin (t)) = (2 cos (t) ; 3 sin (t)) , t ∈ [0; 2π] (1.24) Si costruisce la funzione h, sostituendo in f le componenti di γ:

h (t) = f (2 cos (t) , 3 sin (t)) =x2+ 3y

x = 2 cos (t) y = 3 sin (t)

= 4 cos2(t) + 9 sin (t) (1.25)

Si trovano i punti stazionari di h:

h0(t) = −8 cos (t) sin (t) + 9 cos (t) = cos (t) (9 − 8 sin (t)) = 0 (1.26) Legge di annullamento del prodotto: il primo fattore è pari a zero per:

cos (t) = 0 −→ t = π

2 ∨ t = 3π

2 (1.27)

Il secondo, invece, dà luogo ad un'equazione impossibile:

9 − 8 sin (t) = 0 −→ sin (t) = 9

8 (1.28)

A partire dai due valori di t trovati si ottengono le coordinate:

t = π 2 −→

(x = 2 cos π2 = 0

y = 3 sin π2 = 3 (1.29)

t = 3π 2 −→

(x = 2 cos 2 = 0

y = 3 sin 2 = −3 (1.30)

Due candidati sono quindi P0(0; 3) e P1(0; −3). Si devono considerare inoltre gli estremi del dominio:

t = 0e t = 2π. In corrispondenza di tali valori del parametro si ottengono le coordinate:

t = 0 −→

(x = 2 cos (0) = 2

y = 3 sin (0) = 0 (1.31)

t = 2π −→

(x = 2 cos (2π) = 2

y = 3 sin (2π) = 0 (1.32)

Il terzo candidato è quindi P2(2; 0). Si calcolano la immagini dei punti:

f (P0) = f (0, 3) = 0 + 9 = 9 (1.33)

f (P1) = f (0, −3) = 0 − 9 = −9 (1.34)

f (P2) = f (2, 0) = 22+ 0 = 4 (1.35)

Si può inne costruire la tabella del confronto:

punto x y f (x, y)

P0 0 3 9 M AX

P1 0 −3 −9 min

P2 2 0 4

(1.36)

(5)

3.

f (x, y) = x3+ x2− y + 2 I =(x; y) ∈ R2: y = x2− 1 Il vincolo rappresenta una parabola.

−3 −2 −1 1 2 3

−1 1 2 3 4 5 6 7 8

x y

Figura 1.3: Vincolo.

La funzione è continua ma l'insieme ammissibile NON è chiuso e limitato. Nulla assicura che massimo e minimo assoluti esistano. Si valutano le tre possibili opzioni:

(a) Metodo parametrico: ottimo, il vincolo è una curva facilmente parametrizzabile.

(b) Metodi dei moltiplicatori di Lagrange: ottimo, il vincolo è esprimibile nella forma g (x, y) = y − x2− 1 = 0.

(c) Metodo graco: pessimo, le curve di livello sono funzione non note.

Si propende per il METODO PARAMETRICO.

Si parametrizza la parabola:

γ (t) = t; t2− 1 , t ∈ R (1.37)

Si costruisce la funzione h, sostituendo in f le componenti di γ:

h (t) = f t, t2− 1 = x3+ x2− y + 2

x = t y = t2− 1

= t3+ t2− t2− 1 + 2 = t3+ 1 (1.38)

Si trovano i punti stazionari di h:

h0(t) = 3t2= 0 −→ t = 0 (1.39)

Dal momento che non sussistono altri candidati, l'unico modo per determinare la natura del punto è studiare il segno della derivata:

h0(t) = 3t2≥ 0 sempre (1.40)

Questo signica che t = 0 è un punto di esso, che la restrizione della funzione f al vincolo assegnato è sempre crescente e che quindi non ammette massimo e minimo assoluti.

NOTA: chi avesse utilizzato il metodo dei moltiplicatori di Lagrange si sarebbe trovato in una brutta situazione. La risoluzione del sistema potrebbe infatti indurre a concludere arettatamente che (0; −1) è un punto di estremo. Un breve tuo negli oscuri meandri della teoria, tuttavia, porta alla luce un dettaglio non trascurabile: il metodo di Lagrange ore infatti una condizione necessaria ma non suciente anché un punto sia di estremo.

4.

f (x, y, z) = 3x2+ 2y2+ 4z2 I =(x; y; z) ∈ R3: 2x + 4y − 6z + 5 = 0 Il vincolo rappresenta un piano nello spazio tridimensionale.

(6)

Figura 1.4: Vincolo.

La funzione è continua ma l'insieme ammissibile NON è chiuso e limitato. Nulla assicura che massimo e minimo assoluti esistano. Si valutano le tre possibili opzioni:

(a) Metodo parametrico: pessimo, per rappresentare un piano servono due parametri.

(b) Metodi dei moltiplicatori di Lagrange: ottimo, il vincolo è esprimibile nella forma g (x, y) = 2x + 4y − 6z + 5 = 0.

(c) Metodo graco: pessimo, è un metodo inadatto per funzioni 3D.

Si propende per il METODO DEI MOLTIPLICATORI DI LAGRANGE.

Si costruisce la funzione Lagrangiana:

L (x, y, z, λ) = f (x, y, z) − λg (x, y, z) = 3x2+ 2y2+ 4z2− λ (2x + 4y − 6z + 5) (1.41) Si annulla poi il gradiente di L:









∂L(x,y,z,λ)

∂x = 6x − 2λ = 0

∂L(x,y,z,λ)

∂y = 4y − 4λ = 0

∂L(x,y,z,λ)

∂z = 8z + 6λ = 0

∂L(x,y,z,λ)

∂λ = − (2x + 4y − 6z + 5) = 0

(1.42)

Risolvendo il sistema si trovano i punti di estremo.









x = λ3 =y3 y = λ

z = −4 = −3y4 2x + 4y − 6z + 5 = 0

(1.43)









x = λ3 = y3 y = λ z = 4 = 3y4

2 ·y3 + 4y + 6 ·3y4 + 5 = 0

(1.44)







 x = y3 λ = y z = 3y4

55y 11 = −5

(1.45)









x = −112 y = −116 z = 229 λ = −116

(1.46)

Il punto di estremo da considerare è quindi P −112; −116; −229. Dal momento che la funzione è sempre convessa e non è limitata, è logico pensare che quello trovato sia un punto di minimo e che non ammetta massimo sull'insieme illimitato individuato dal piano.

(7)

5.

f (x, y) = xy − y2+ 3 I =(x; y) ∈ R3: −1 ≤ x ≤ 1 − y2

Il vincolo è espresso da un sistema di disequazioni, che individuano la regione di piano compresa tra la retta verticale x = −1 e la parabola x = 1 − y2. L'insieme ammissibile è quindi chiuso e limitato e la funzione è continua: l'esistenza di minimo e massimo è assicurata dal teorema di Weierstrass.

−2.0 −1.5 −1.0 −0.5 0.5 1.0 1.5 2.0

−2

−1 1 2

x y

Figura 1.5: Vincolo.

Occorre dapprima considerare i punti interni all'insieme, applicando il consueto criterio per la determina- zione dei punti critici:

∇f (x, y) = (y; x − 2y) = (0; 0) (1.47)

Si ricava che l'unica soluzione è P0(0; 0): dal momento che è un punto interno all'insieme, diviene il primo candidato per il confronto nale.

Si passa poi ad analizzare la frontiera. Si valutano le tre possibili opzioni:

(a) Metodo parametrico: ottimo, il vincolo è l'unione di curve facilmente parametrizzabili.

(b) Metodi dei moltiplicatori di Lagrange: pessimo, il vincolo non è esprimibile nella forma g (x, y) = 0.

(c) Metodo graco: pessimo, le curve di livello sono funzione non note.

Si propende per il METODO PARAMETRICO.

Per parametrizzare il segmento di retta x = −1 è necessario conoscere i suoi estremi, ovvero i punti di intersezione con la parabola:

(x = −1

x = 1 − y2 (1.48)

(x = −1

−1 = 1 − y2−→ y2= 2 −→ y = ±√

2 (1.49)

Di conseguenza la curva risulta essere:

γ (t) = (−1; t) , t ∈h

−√ 2;√

2i

(1.50) Si costruisce la funzione h, sostituendo in f le componenti di γ:

h (t) = f (−1, t) =xy − y2+ 3

x = −1 y = t

= −t − t2+ 3 (1.51)

Si trovano i punti stazionari di h:

h0(t) = 2t − 1 = 0 −→ t = −1

2 (1.52)

Che corrisponde alle coordinate:

(x = −1

y = t = −12 (1.53)

(8)

Il secondo candidato è quindi P1 −1; −12. Si considerano anche gli estremi del dominio, ovvero i punti corrispondenti a t = ±√

2: P2 −1; −√

2e P3 −1;√ 2. Si passa poi a considerare l'arco di parabola:

γ (t) = 1 − t2; t , t ∈h

−√ 2;√

2i

(1.54) Si costruisce la funzione h, sostituendo in f le componenti di γ:

h (t) = f 1 − t2, t = xy − y2+ 3

x = 1 − t2 y = t

= t − t3− t2+ 3 (1.55)

Si trovano i punti stazionari di h:

h0(t) = −3t2− 2t + 1 = 0 −→ 3t2+ 2t − 1 = 0 (1.56) t4,5= −1 ±√

1 + 3

3 −→ t = 1

3∨ t = −1 (1.57)

Che corrispondono alle coordinate:

t = −1 −→

(x = 1 − t2= 0

y = t = −1 (1.58)

t = 1 3 −→

(x = 1 − t2=89

y = t = 13 (1.59)

Gli ultimi candidati sono quindi P4(0; −1) e P5 8

9;13; gli estremi del dominio corrispondono infatti ai punti P3 e P2. Si può ora costruire la tabella del confronto:

punto x y f (x, y)

P0 0 0 3

P1 −1 −12 134 M AX P2 −1 −√

2 √

2 + 1

P3 −1 √

2 1 −√

2 min

P4 0 −1 2

P5 89 13 8627

(1.60)

6.

f (x, y) = xy I =(x; y) ∈ R2: x4+ y4≤ 1

Il vincolo è espresso da una disequazione che individua una regione di piano compresa all'interno di una curva sconosciuta. L'insieme ammissibile è quindi chiuso e limitato e la funzione è continua: l'esistenza di minimo e massimo è assicurata dal teorema di Weierstrass.

−2.0 −1.5 −1.0 −0.5 0.5 1.0 1.5 2.0

−2

−1 1 2

x y

Figura 1.6: Vincolo.

Occorre dapprima considerare i punti interni all'insieme, applicando il consueto criterio per la determina- zione dei punti critici:

∇f (x, y) = (y; x) = (0; 0) (1.61)

L'equazione è risolta solo da P0(0; 0): il primo candidato è quindi l'origine degli assi.

Si passa poi ad analizzare la frontiera. Si valutano le tre possibili opzioni:

(9)

(a) Metodo parametrico: pessimo, il vincolo non è una curva conosciuta.

(b) Metodi dei moltiplicatori di Lagrange: ottimo, il vincolo è esprimibile nella forma g (x, y) = x4+ y4− 1 = 0.

(c) Metodo graco: pessimo, le curve di livello sono iperboli.

Si propende per il METODO DEI MOLTIPLICATORI DI LAGRANGE.

Si costruisce la funzione Lagrangiana:

L (x, y, λ) = f (x, y) − λg (x, y) = xy − λ x4+ y4− 1

(1.62) Si annulla poi il gradiente di L:





∂L(x,y,λ)

∂x = x − 4λx3= 0

∂L(x,y,λ)

∂y = y − 4λy3= 0

∂L(x,y,λ)

∂λ = − x4+ y4− 1 = 0

(1.63)

Risolvendo il sistema si trovano i punti di estremo.





x 1 − 4λx2 = 0 y 1 − 4λy2 = 0 x4+ y4− 1 = 0

(1.64)

La prima equazione è risolta per x = 0 o 1 − 4λx2= 0. Nel primo caso il sistema diventa:



 x = 0

y 1 − 4λy2 = 0 y4= 1 −→ y = ±1

(1.65)



 x = 0

±1 (1 − 4λ) = 0 −→ λ = 14 y = ±1

(1.66)



 x = 0 y = ±1 λ = 14

(1.67)

Nel secondo caso il sistema diviene:





1 − 4λx2= 0 −→ λ = 4x12

y 1 − 4λy2 = 0 −→ y 1 − 44x12y2 = 0 −→ y

1 − yx22

= 0 x4+ y4= 1

(1.68)





 λ = 4x12

y 1 − yx22



= 0 −→ y = 0 ∨ y2= x2 x4+ y4= 1

(1.69)

Si è potuto ipotizzare x 6= 0 perché questa evenienza è già stata presa in considerazione nel primo caso.

A questo punto, se y = 0, l'ultima equazione fornisce x = ±1: le soluzioni del sistema sono quindi:



 x = ±1 y = 0 λ = 14

(1.70)

Se invece y2= x2l'ultima equazione è risolta per 2x4= 1 −→ x = ±4 q1

2. Le soluzioni sono:





 x = +4

q1 2

y = ±q4

1 2

λ = 212

(1.71)

(10)





x = −q4

1 2

y = ±4 q1

2

λ = 1

2 2

(1.72)

Ricapitolando, i punti da inserire nel confronto sono: (0; ±1), (±1; 0), +q4

1 2; ±q4

1

2, −q4

1 2; ±q4

1

2. Si esegue inne il confronto:

punto x y f (x, y)

P0 0 0 0

P1 0 1 0

P2 0 −1 0

P3 −1 0 0

P4 1 0 0

P5 4

q1 2

4

q1 2

q1

2 M AX

P6 4

q1 2 −q4

1

2 −q

1

2 min

P7 −q4

1 2

4

q1

2 −q

1

2 min

P8 −q4

1 2 −q4

1 2

q1

2 M AX

(1.73)

Determinare per via graca gli eventuali punti di massimo e minimo della funzione f soggetta al vincolo I.

1.

f (x, y) = x2+ y2+ 1 I =(x; y) ∈ R2: |x| + |y| = 1

Per utilizzare il metodo graco occorre innanzitutto disegnare l'insieme descritto dal vincolo:

|x| + |y| = 1 ⇐⇒

(y = 1 − |x| y ≥ 0

y = |x| − 1 y < 0 (1.74)

Si ottiene un quadrato di lato 1 e centro di simmetria nell'origine.

−1.0 −0.8 −0.6 −0.4 −0.2 0.2 0.4 0.6 0.8 1.0

−1.0

−0.8

−0.6

−0.4

−0.2 0.2 0.4 0.6 0.8 1.0

x y

Figura 1.7: I = (x; y) ∈ R2: |x| + |y| = 1 .

Si devono quindi determinare le curve di livello della funzione:

f (x, y) = x2+ y2+ 1 = l (1.75)

f (x, y) = x2+ y2= l − 1 (1.76)

Si riconoscono circonferenza centrate nell'origine di raggio r =√

l − 1con l ≥ 1.

(11)

-3 -2 -1 0 1 2 3 -3

-2 -1 0 1 2 3

2 4 6 8 10 12 14 16 18

2 0 -2 -2

0 2 15

10

5

2 4 6 8 10 12 14 16 18

Figura 1.8: Curve di livello.

Al crescere di l il raggio aumenta: le curve di livello più basso sono quindi le circonferenze di raggio minore.

Inserendo nello stesso graco il vincolo e le curve di livello, si nota come la più piccola circonferenza che interseca il vincolo sia quella inscritta nel quadrato, mentre la più grande è quella circoscritta, ovvero quella passante per i vertici. Si conclude quindi che i punti di tangenza tra circonferenza inscritta e quadrato, ovvero i punti medi dei lati, sono di minimo assoluto, mentre i vertici sono di massimo assoluto.

-3 -2 -1 0 1 2 3

-3 -2 -1 0 1 2 3

2 4 6 8 10 12 14 16 18

Figura 1.9: Vincolo e curve di livello.

punto x y f (x, y)

P1 0 1 2 M AX

P2 0 −1 2 M AX

P3 −1 0 2 M AX

P4 1 0 2 M AX

P5 12 12 32 min

P6 1212 32 min P712 12 32 min P81212 32 min

(1.77)

(12)

1.2 Problemi di massimo e minimo.

Risolvere i seguenti problemi di massimo e minimo.

1. Tra tutti i parallelepipedi di area totale A, trovare le dimensioni di quello di volume massimo.

Per risolvere un problema di ottimizzazione vincolata occorre determinare tre elementi:

(a) le incognite sono le dimensioni spaziali del parallelepipedo: x, y e z;

(b) la funzione da ottimizzare è in questo caso il volume del parallelepipedo date le dimensioni.

Utilizzando le note formule di geometria solida si ricava f (x, y, z) = xyz;

(c) il vincolo è imposto dall'area del solido. Questa si ottiene sommando le aree delle facce del parallele- pipedo, ricordando che quelle opposte sono isometriche: I = (x; y; z) ∈ R3: 2xy + 2yz + 2xz = A Occorre inoltre considerare il fatto che le incognite rappresentano segmenti: devono quindi assumere. valori positivi. In denitiva:

I =(x; y; z) ∈ R3: 2xy + 2yz + 2xz = A ∧ x > 0 ∧ y > 0 ∧ z > 0

(1.78) Si può ora procedere: dato che la funzione presenta 3 variabili, si propende per il METODO DEI MOLTIPLICATORI DI LAGRANGE. Si costruisce la funzione Lagrangiana:

L (x, y, λ) = f (x, y, z) − λg (x, y, z) = xyz − λ (2xy + 2yz + 2xz − A) (1.79) Si annulla poi il gradiente di L:









∂L(x,y,z,λ)

∂x = yz − 2λy − 2λz = 0

∂L(x,y,z,λ)

∂y = xz − 2λx − 2λz = 0

∂L(x,y,z,λ)

∂z = xy − 2λx − 2λy = 0

∂L(x,y,z,λ)

∂λ = − (2xy + 2yz + 2xz − A) = 0

(1.80)

Risolvendo il sistema si trovano i punti di estremo.









yz − 2λy − 2λz = 0 xz − 2λx − 2λz = 0 xy − 2λx − 2λy = 0 2xy + 2yz + 2xz − A = 0

(1.81)

Sottraendo membro a membro la prima e la seconda equazione e la seconda e la terza equazione si ottengono due relazioni equivalenti, che prendono il posto di altre due del sistema precedente.









(yz − 2λy − 2λz) − (xz − 2λx − 2λz) = 0 (xz − 2λx − 2λz) − (xy − 2λx − 2λy) yz − 2λy − 2λz = 0

2xy + 2yz + 2xz − A = 0

(1.82)









yz − 2λy − xz + 2λx = 0 xz − 2λz − xy + 2λy = 0 yz − 2λy − 2λz = 0 2xy + 2yz + 2xz − A = 0

(1.83)









z (y − x) − 2λ (y − x) = 0 x (z − y) − 2λ (z − y) = 0 yz − 2λy − 2λz = 0 2xy + 2yz + 2xz − A = 0

(1.84)









(z − 2λ) (y − x) = 0 (x − 2λ) (z − y) = 0 yz − 2λy − 2λz = 0 2xy + 2yz + 2xz − A = 0

(1.85)

(13)

La prima equazione è risolta se z = 2λ oppure se y = x. Ipotizzando z = 2λ si ha:







 z = 2λ

(x − 2λ) (2λ − y) = 0

2yλ − 2λy − 4λ2= 0 −→ λ = 0 2xy + 4yλ + 4xλ = A

(1.86)









z = 2λ = 0 xy = 0 λ = 0 2xy = A

(1.87)

Il sistema risulta quindi impossibile. Non rimane che tentare y = x.







 y = x

(x − 2λ) (z − x) = 0 xz − 2λx − 2λz = 0 2x2+ 2xz + 2xz = A

(1.88)

La seconda equazione è risolta in due casi: accettando x = 2λ si ricadrebbe in una situazione analoga al caso z = 2λ, quindi si considera z = x.







 y = x z = x

x2− 2λx − 2λx = 0 2x2+ 2x2+ 2x2= A

(1.89)







 y = x z = x

x (x − 4λ) = 0 6x2= A

−→













y = x =q

A 6

z = x = qA

6

λ = x4 x =

qA 6

(1.90)

Il solido che massimizza il volume è quindi un cubo di latoq

A

6. Si giustica il fatto che il punto trovato corrisponda ad un massimo con la seguente considerazione. Facendo tendere a 0 una delle dimensioni, si ottiene un solido che tende sempre più ad una gura piana. Al limite il volume tende quindi a zero, senza però poter mai raggiungere questo valore, a causa del vincolo iniziale. In altre parole, la funzione che rappresenta il volume del solido non ammette minimo.

2. Tra tutti i parallelepipedi di volume V , trovare le dimensioni di quello di area minima.

Per risolvere un problema di ottimizzazione vincolata occorre determinare tre elementi:

(a) le incognite sono le dimensioni spaziali del parallelepipedo: x, y e z;

(b) la funzione da ottimizzare è in questo caso l'area del parallelepipedo date le dimensioni. Que- sta si ottiene sommando le aree delle facce del parallelepipedo, ricordando che quelle opposte sono isometriche: f (x, y, z) = 2xy + 2yz + 2xz;

(c) il vincolo è imposto dal volume del solido, ovvero dal prodotto delle tre dimensioni I = (x; y; z) ∈ R3: xyz = V Occorre inoltre considerare il fatto che le incognite rappresentano segmenti: devono quindi assumere .

valori positivi. In denitiva:

I =(x; y; z) ∈ R3: xyz = V ∧ x > 0 ∧ y > 0 ∧ z > 0 (1.91) Si può ora procedere: dato che la funzione presenta 3 variabili, si propende per il METODO DEI MOLTIPLICATORI DI LAGRANGE. Si costruisce la funzione Lagrangiana:

L (x, y, λ) = f (x, y, z) − λg (x, y, z) = 2xy + 2yz + 2xz − λ (xyz − V ) (1.92)

(14)

Si annulla poi il gradiente di L:









∂L(x,y,z,λ)

∂x = 2y + 2z − λyz = 0

∂L(x,y,z,λ)

∂y = 2x + 2z − λxz = 0

∂L(x,y,z,λ)

∂z = 2y + 2x − λxy = 0

∂L(x,y,z,λ)

∂λ = − (xyz − V ) = 0

(1.93)

Risolvendo il sistema si trovano i punti di estremo.









2y + 2z − λyz = 0 2x + 2z − λxz = 0 2y + 2x − λxy = 0 xyz = V

(1.94)

Si ricava λ dalla prima equazione e lo si sostituisce nelle altre:









2y + 2z = λyz −→ λ = 2y+2zyz 2x + 2z − 2y+2zyz xz = 0 2y + 2x −2y+2zyz xy = 0 xyz = V

(1.95)









λ = 2y+2zyz

2yx + 2yz − 2yx − 2xz = 0 2zy + 2zx − 2xy − 2xz = 0 xyz = V

(1.96)









λ = 2y+2zyz

yz − xz = 0 −→ y = x 2zy − 2xy = 0 −→ z = x xyz = V

(1.97)









λ = 2x+2xx2 = 4x y = x

z = x x3= V

(1.98)









λ = x4 = 34

V

y = x =√3 V z = x =√3

V x =√3

V

(1.99)

Il solido che minimizza l'area è quindi un cubo di lato √3

V. Si giustica il fatto che il punto trovato corrisponda ad un massimo con la seguente considerazione. Facendo tendere a 0 una delle dimensioni, si ottiene un solido che tende sempre più ad una gura piana. Per fare sì che il volume rimanga costante, le altre dimensioni devono aumentare indenitamente. In altre parole, l'area del parallelepipedo non è una funzione superiormente limitata.

Riferimenti

Documenti correlati

Per r &gt; R, si utilizza lo stesso procedimento, tenendo in conto che la distanza dal centro aumenta, ma la carica interna alla supercie attraverso la quale si calcola il usso

quale inclinazione deve avere F 2 perché la rotta della chiatta sia parallela alla riva;. lo spazio percorso dalla chiatta, partita da ferma, prima di raggiungere la velocità v =

Se invece si imprimesse una forza molto intensa, il corpo tenderebbe a scorrere verso l'alto, e subirebbe un attrito di verso opposto.. 2.5

Sapendo che il coeciente d'attrito dinamico tra la neve e lo slittino aumenta lungo il pendio secondo l'equazione µ(x) = ax 3 2 + b calcolare il minimo lavoro che il ragazzo

Due dischi di uguale massa m e raggio r sono saldati agli estremi di un'asta omogenea orizzontale di lunghezza 2r e massa m a che e' posta in rotazione attorno ad un asse

Due dischi di uguale massa m e raggio r sono saldati agli estremi di un'asta omogenea orizzontale di lunghezza 2r e massa m a che e' posta in rotazione attorno ad un asse

La prima asta può ruotare senza attrito attorno ad un asse orizzontale passante per il punto O, mentre la seconda è libera di ruotare senza attrito attorno ad un asse

La prima asta può ruotare senza attrito attorno ad un asse orizzontale passante per il punto O, mentre la seconda è libera di ruotare senza attrito attorno ad un asse